Logeometric progression

Algebra Level 3

{ a + log 2 3 , a + log 4 3 , a + log 8 3 } \{ a + \log_2 3, a + \log_4 3, a + \log_8 3 \}

Let a a be a real number such that the above set of numbers form a geometric progression (in that order). Find the common ratio of this geometric progression.


The answer is 0.33.

This section requires Javascript.
You are seeing this because something didn't load right. We suggest you, (a) try refreshing the page, (b) enabling javascript if it is disabled on your browser and, finally, (c) loading the non-javascript version of this page . We're sorry about the hassle.

2 solutions

Peter Macgregor
Apr 13, 2015

I came at it from a slightly different angle.

Since the three terms are in G.P. we can write

r ( a + log 2 3 ) = a + log 4 3 r(a+\log_2 3)=a+\log_4 3

r ( a + log 4 3 ) = a + log 8 3 r(a+\log_4 3)=a+\log_8 3

Subtracting the equations eliminates a to give

r ( log 2 3 log 4 3 ) = log 4 3 log 8 3 r(\log_2 3-\log_4 3)=\log_4 3-\log_8 3

Now change all the logarithms to base 2 to get

r ( log 2 3 log 2 3 2 ) = log 2 3 2 log 2 3 3 r\left(\log_2 3-\dfrac{\log_2 3}{2}\right)=\dfrac{\log_2 3}{2}-\dfrac{\log_2 3}{3}

Now the logarithm can be cancelled as a factor leaving

r 2 = 1 6 \dfrac{r}{2}=\dfrac{1}{6}

and so

r = 1 3 \boxed{r=\dfrac{1}3{}}

Let x = log 2 ( 3 ) . x = \log_{2}(3). Then by the change of base rule the progression becomes

a + x , a + x 2 , a + x 3 . a + x, a + \dfrac{x}{2}, a + \dfrac{x}{3}.

As the progression is geometric, we must have that

a + x 2 a + x = a + x 3 a + x 2 a 2 + a x + x 2 4 = a 2 + 4 a x 3 + x 2 3 \dfrac{a + \dfrac{x}{2}}{a + x} = \dfrac{a + \dfrac{x}{3}}{a + \dfrac{x}{2}} \Longrightarrow a^{2} + ax + \dfrac{x^{2}}{4} = a^{2} + \dfrac{4ax}{3} + \dfrac{x^{2}}{3}

a x 3 = x 2 12 4 a x = x 2 . \Longrightarrow -\dfrac{ax}{3} = \dfrac{x^{2}}{12} \Longrightarrow -4ax = x^{2}.

Now clearly x 0 , x \ne 0, so a = x 4 . a = -\dfrac{x}{4}. The desired ratio is then

x 4 + x 2 x 4 + x = x 4 3 x 4 = 1 3 = 0.333 \dfrac{-\dfrac{x}{4} + \dfrac{x}{2}}{-\dfrac{x}{4} + x} = \dfrac{\dfrac{x}{4}}{\dfrac{3x}{4}} = \dfrac{1}{3} = \boxed{0.333} to 3 decimal places.

Same method!

Adarsh Kumar - 6 years, 2 months ago

Hey i typed the answer 0.33 but it showed incorrect answer

Raghav Rathi - 4 years, 10 months ago

I calculated the correct answer but the answer field would not take the fraction. There is nothing in the statement to suggest a decimal approximation. So when I looked st the solutions I think I was scored as if I missed the problem. Not fair!!!

Will Heierman - 4 years, 5 months ago

Log in to reply

The answer field never accepts fractions such as 1 / 3 1/3 . In cases where the answer is not an integer, there is a small caption under the answer field that says "decimals o.k.", and in this case 0.33 0.33 or 0.333 0.333 should be accepted as correct. That being said, Raghav Rathi mentioned below that he entered this value only to be told he was incorrect. Is that what happened to you too?

Brian Charlesworth - 4 years, 5 months ago

At first I took it to mean that the terms given were meant to suggest a general term, which was not the case. There is no "..." there.

Not only it is not a general term, the geometric progression given terminates (as far as being geometric is concerned) one term past the three given terms at 0 when it cannot be continued by a common ratio.

I missed Macgregor's shortcut to the ratio without bothering about a at all. Very nice.

Nonetheless I found the reduction log2^k(3) = (1/k) log2(3), rewriting the sequence as {a + (1/k)log2(3)}.

I then managed to find the 1/3 ratio solving using the first three terms much like the Charlesworth solution that solves for "a" getting a =-x/4 which is actually a = -log2(3)/4.

The problem is that after the first three wonderful terms 3log2(3)/4, log2(3)/4, log2(3)/12 that yield the common ratio 1/3 just fine, there is -log2(3)/4 + (1/4)log2(3) = 0. So much for the common ratio. A progression but not an infinite sequence as seen by jumping past that to terms 5 and 6.

(-log2(3)/4 + (1/6)log2(3)) / (-log2(3)/4 + (1/5)log2(3)) = (-1/12) / (-1/20) = 20/12 = 5/3 not 1/3.

Can one change a harmonic sequence with ratio k/k+1 into a geometric sequence by mere translation by a constant for more than three terms as seen here? A new quiz question.

If one abandons the form of the first three terms as a general term, there no reason it cannot be extended indefinitely with multiplication by 1/3 thereafter starting from the third term. Any two positive numbers in a prescribed order define an infinite geometric progression using the common ratio not unlike the way two points determine a line. It is only that whatever is used to generate the initial terms may not be a general term for a geometric sequence.

Robert DeLisle - 4 years, 3 months ago

0 pending reports

×

Problem Loading...

Note Loading...

Set Loading...